Valor esperado del operador de posición para estados de momento angular

Tengo un ejercicio donde se pide probar que este elemento de la matriz:

j = 2 , METRO = 1 | X ^ 2 ^ X | j = 2 , METRO = 1

es 0

El problema es que no sé cómo actúan los operadores de espacio y momento en los estados de momento angular. ¿Alguna idea de cómo afrontar este problema?

Probablemente haya algún tipo de pista de simetría ;)

Respuestas (2)

Si PAG es el operador de paridad unitario y autoadjunto, es bien sabido que PAG | j , metro = ( 1 ) j | j , metro mientras

PAG X k = X k PAG y PAG PAG k = PAG k PAG
para k = 1 , 2 , 3 con X k Operador de posición con respecto al k -ésimo eje y PAG k el operador de cantidad de movimiento análogo.

El texto del ejercicio en realidad está usando un formalismo un poco impropio como | j , metro solo especifica la parte del vector en L 2 ( S 2 , d Ω ) pero nada se dice refiriéndose a la parte radial: otro número cuántico norte en cuanto a la variable radial, | norte , j , metro , debe agregarse, pero no importa, ya que no invalida la identidad escrita anteriormente PAG | norte , j , metro = ( 1 ) j | norte , j , metro

El operador que aparece en tu bra-ket es O = X 1 2 PAG 1 hasta factores numéricos (y sutilezas con dominios que ignoraré aquí) de modo que,

PAG O = O PAG .
Finalmente, con eso O ,
j , metro | O | j , metro = ( 1 ) j j , metro | O PAG | j , metro = ( 1 ) j ( 1 ) j , metro | PAG O | j , metro = ( 1 ) j ( 1 ) ( 1 ) j j , metro | O | j , metro = j , metro | O | j , metro .
Resumiendo
j , metro | O | j , metro = j , metro | O | j , metro
de modo que
j , metro | O | j , metro = 0
como quería

Obsérvese que si escribiésemos explícitamente el número cuántico radial, obtendríamos una identidad más precisa

norte , j , metro | O | norte , j , metro = 0
a lo largo del mismo argumento proporcionado j + j incluso. Insisto en que es posible que tengamos norte norte y metro metro arriba.

Como observación final, no es correcto decir que este es el valor esperado de O desde O no es autoadjunto (ni hermitiano).

¡Muchas gracias! Hemos hecho simetría de paridad durante el curso para que todo tenga sentido.

Primero debe convertir sus estados de momento angular en armónicos esféricos y luego de armónicos esféricos a coordenadas cartesianas. El problema no menciona ninguna parte radial asi que supongo que sera algo generico F ( r ) .

Alternativamente, puede convertir / X y X 2 a coordenadas esféricas y luego hacer que actúen en los armónicos esféricos apropiados.

En ambos casos es necesario convertir

| , metro F ( r ) Y , metro ( θ , ϕ ) .